Каким будет эффективное сопротивление лестницы из резисторов, состоящей из n ступеней?

Я репетитор. Это проблема школьного уровня. В старших классах каждый мог решить задачу об эффективном сопротивлении лестницы резисторов с бесконечными ступенями. Теперь проблема немного другая. что, если у него есть nшаги вместо бесконечных шагов. Как рассчитать эффективное сопротивление в этом случае?введите описание изображения здесь

Ответы (6)

Я сделаю это шаг за шагом здесь. Сначала я напишу ответ для первых нескольких случаев с анализом схемы. Затем я применю сокращение, чтобы показать шаблон, к которому приходит проблема.

Н=1

Z знак равно р + р знак равно 2 р

N=2

Z знак равно р + 1 1 р + 1 р + р знак равно р ( 1 + 1 1 + 1 1 + 1 ) знак равно 5 3 р

N=3

Z знак равно р + 1 1 р + 1 р + 1 1 р + 1 р + р знак равно р ( 1 + 1 1 + 1 1 + 1 1 + 1 1 + 1 ) знак равно 13 8 р

В этот момент цепная дробь четко различима. Я нахожу это немного тревожным, потому что он не масштабируется с Н , но непрерывная дробь растет как 2 Н . Это можно исправить, не рассматривая возможность добавления 2 резисторов на конце каждый раз, а вместо этого добавляя один резистор параллельно, один последовательно к этому, затем новый параллельно между двумя новыми и используя для этого другой индекс.

Вычислительные программы должны иметь простую функцию для записи непрерывной дроби в некоторые Н количество фракций. К сожалению я не могу найти это для Maple, но мне нужна такая процедура, чтобы дать ответ на ваш вопрос. Я дам определение такой вещи прямо здесь. я намеренно использую н в определении и не Н во избежание неизбежной путаницы.

Ф ( 1 ) знак равно 1
Ф ( н + 1 ) знак равно 1 + 1 Ф ( н )

Этим я могу ответить на ваш вопрос.

Z знак равно р Ф ( 2 Н )

И я могу дать вам пример функции, которую я только что создал.

Ф ( 1..6 ) знак равно [ 1 , 2 , 3 2 , 5 3 , 8 5 , 13 8 ]
Ф ( ) знак равно 5 + 1 2

Я думаю, что это лучшая форма, с помощью которой можно решить проблему. Дробь не может быть легко приведена к какой-либо краткой алгебраической форме, потому что весь смысл этого упражнения состоит в том, чтобы не вводить допущений, а большая дробь является алгебраической формой ответа. Однако конечные значения дроби настолько легко реализовать процедурно, насколько это возможно.

Решение закрытой формы

Это должно быть мое последнее редактирование, и это выражение в значительной степени решает проблему.

Ф ( н ) знак равно ф н + 1 ( 1 ф ) н + 1 ф н ( 1 ф ) н
ф знак равно 5 + 1 2
Z знак равно р ф 2 Н + 1 ( 1 ф ) 2 Н + 1 ф 2 Н ( 1 ф ) 2 Н

Стоит прямо сказать, что Ф ( н ) есть отношение (n+1)-го и n-го чисел Фибоначчи . Другими словами, у вас есть Ф ( н ) знак равно Ф я б ( н + 1 ) Ф я б ( н ) ).
@Спенсер Точно. Я сдерживался, так как это уже было в ответе Теда Банна.

Для любого заданного н , вы можете решить это с помощью правил для последовательных и параллельных резисторов, но чтобы получить общую формулу, действительную для всех н , мне не кажется легким. Лучший способ, который я знаю, - это получить рекурсивное отношение, дающее сопротивление н - стремянка в плане ( н 1 ) -стремянка. Если я не ошибаюсь, то н -лестницу можно рассматривать как одиночный резистор р последовательно с параллельной комбинацией, состоящей из другого р и ( н 1 ) -стремянка. Правила последовательного и параллельного сопротивления дают

р н знак равно р + р р н 1 р + р н 1 .
Начальная одноступенчатая лестница имеет р 1 знак равно 2 р , и вы можете использовать приведенную выше формулу, чтобы двигаться вверх по цепочке оттуда.

Я заставил Mathematica проработать первые шаги, и они, кажется, связаны с числами Фибоначчи, в частности

р н знак равно р Ф 2 н + 1 Ф 2 н .
Я уверен, что это можно доказать по индукции.

Я получил тот же результат, см. мою окончательную форму Ф ( н ) . Существуют всевозможные способы получить результат, но формулирование проблемы в виде ряда (как это сделано здесь в нескольких ответах) приведет вас к последовательности, которая может быть математически показана как отношение н + 1 к н Числа Фибоначчи, как вы нашли. Хорошая работа, делающая это наблюдение.

Интересно. Я думаю, вы хотите действовать следующим образом: (1) Без ограничения общности предположим, что входное напряжение равно 1, а резисторы имеют один Ом. (2) Определить r = сопротивление бесконечной лестницы. (3) Теперь посмотрите на схему, где мы добавляем еще один шаг с левой стороны. Мы должны найти напряжение там, где оно подключено, как функцию r. (4) Новый ток равен 1 минус это напряжение. А полное сопротивление конфигурации r'=1/(1-V(r)). (5) Скорректируйте r так, чтобы r=r' (6) Масштабируйте по R, указанному в исходной задаче. Т.е. результат r*R.

Немного алгебры в шагах 4 и 5.....

Работайте в обратном направлении, чтобы упростить математику, а также ускорить появление шаблона. Я сделал это, но покажу результаты в прямом порядке

Р(1) = 2Р = (1+1/1)Р

R(2) = (1+(2/3)) R

Р(3) = (1+(5/8)) Р

R(4) = (1+(13/21)) R

R(5) = (1+(34/55)) R

Назовем эту общую форму ---> R(N) = (1+((a(N)/b(N))) R

Определите a(1) = b(1) = 1.

Для N > 1 --->

а (N) = а (N-1) + б (N-1)

б(N) = а(N) + б(N-1) = а(N-1) + 2 б(N-1)

Вам все еще нужно провернуть математику для каждого R (N), так как нужно знать a (N-1) и b (N-1), чтобы получить следующий член; но это более прямолинейно. Может быть, кто-то, разбирающийся в математике сериалов, сможет передать это нам в закрытой форме.

Вы встали на путь, который я бы использовал, чтобы преподавать эту проблему. Начать с последнего добавленного, а затем повторно использовать ответ на каждом последующем шаге схемы, чтобы получить серию, — это самый простой способ решить проблему. Я считаю, что получил закрытую форму, но она не более особенная, чем наблюдение из ответа Теда Банна о том, что ряд состоит из отношений чисел Фибоначчи.

Вот как электрик решает проблему:

Позволять U быть напряжением на входе лестницы резисторов. Позволять я к быть текущим в горизонтальном плече k-го сечения, я 0 быть текущим в горизонтальном плече 0-й секции (начальной секции), я н быть текущим в горизонтальном плече n-й секции (последняя секция). Итак, я работаю на самом деле с н + 1 разделы. Применяя последовательно к замкнутым цепям закон Кирхгофа для напряжений, находим:

р я 0 + р ( я 0 я 1 ) знак равно U

р ( я 0 я 1 ) р я 1 р ( я 1 я 2 ) знак равно 0

. . . . . . . . . . . . . . . . . . . . . . . . . . . . . .

р ( я н 1 я н ) + 2 р я н знак равно 0
После упрощения:

2 я 0 я 1 знак равно U р

3 я к я к + 1 я к 1 знак равно 0

3 я н я н 1 знак равно 0
Теперь эффективное сопротивление:

р е знак равно U я 0
Следующим шагом является решение рекуррентного соотношения. Удивительно, но Wolfram Alpha неплохо справляется со своей задачей: Ссылка здесь . Есть ф ( к ) вместо я к

Так р е знак равно р ( 7 3 5 ) ( 3 5 ) н + ( 3 + 5 ) н + 1 ( 1 + 5 ) ( 3 + 5 ) н 2 ( 5 2 ) ( 3 5 ) н

Этот ответ согласуется с ответом Zassounotsukushi.

Это эквивалентно работе с 1/R = проводимость вместо сопротивления, не так ли?

возьмите только первые 2 резистора и остальные как Икс теперь вертикальный резистор и ваш Икс будет параллельно, эффективное сопротивление будет р Икс / р + Икс с последовательным горизонтальным резистором.

Теперь эквивалентное сопротивление будет

р е д . знак равно р + ( р Икс / р + Икс ) .

  • взять Рек. в качестве Икс опять таки

Составьте квадратное уравнение и решите его Икс . Это будет ответ.